subject
Physics, 19.03.2021 03:20 diegovaldes25

A flat loop of wire consisting of a single turn of cross-sectional area 8.80 cm2 is perpendicular to a magnetic field that increases uniformly in magnitude from 0.500 T to 1.80 T in 1.10 s. What is the resulting induced current if the loop has a resistance of 2.20

ansver
Answers: 1

Another question on Physics

question
Physics, 21.06.2019 20:30
Protons and neutrons are found within the nucleus of an atom
Answers: 2
question
Physics, 22.06.2019 00:30
Next a skier is pulled by a tow rope up a frictionless ski slope that makes an angle of 15 with the horizontal. the rope moves parallel to the slope with a constant speed of 0.69 m/s. the force of the rope does 800 3 of work on the skier as the skier moves a distance of 8.4 m up the incline. (a) if the rope moved with a constant speed of 2.2 m/s how much work would the force of the rope do on the skier as the skler moved a distance of 8.4 m up the incline? at what rate is the force of the rope doing work on the skier when the rope moves with a speed of (b) 0.69 m/s and (c) 2.2 m/s?
Answers: 1
question
Physics, 22.06.2019 02:30
Herbivores, carnivores, and omnivores are all types a. decomposers b. producers c. consumers d. biomes
Answers: 2
question
Physics, 22.06.2019 09:30
Along the line connecting the two charges, at what distance from the charge q1 is the total electric field from the two charges zero? express your answer in terms of some or all of the variables s, q1, q2 and k =14ï€ï¡0. if your answer is difficult to enter, consider simplifying it, as it can be made relatively simple with some work.
Answers: 3
You know the right answer?
A flat loop of wire consisting of a single turn of cross-sectional area 8.80 cm2 is perpendicular to...
Questions
question
Mathematics, 20.01.2021 23:00
question
Mathematics, 20.01.2021 23:00
question
History, 20.01.2021 23:00
Questions on the website: 13722363